Can you help me solve my mathematical problem?

In summary, the conversation discusses the Larmor Equation and its derivation. The speaker has derived an equation that simplifies the original derivation by taking the dot product of both sides with the unit vector. However, they later realize that they needed the original derivation without the dot product to properly understand the equation. They also mention a modification to the Larmor energy and its correct derivation. The conversation ends with the speaker questioning if their understanding is correct.
  • #1
Meselwulf
126
0
I'm definitely messing up somewhere :P

The Larmor Equation is

[tex]\Delta H = \frac{2\mu}{\hbar Mc^2 e} \frac{\partial V(r_{ij})}{\partial r_{ij}} ( L \cdot S)[/tex]

I derived an equation stated as

[tex]\nabla \times \vec{F}_{ij} \cdot \hat{n} = \nabla \times \frac{\partial V(r_{ij})}{\partial r_{ij}}[/tex]

How is put in a simpler way from the original derivation, is imagine we have what we began with

[tex]\vec{F}_{ij} = \frac{\partial V(r_{ij})}{\partial r_{ij}} \hat{n}[/tex]

Take the dot product of the unit vector on both sides gives

[tex]\vec{F}_{ij} \cdot \hat{n} = \frac{\partial V(r_{ij})}{\partial r_{ij}}[/tex]

Now taking the curl of F is

[tex]\nabla \times \vec{F}_{ij} \cdot \hat{n} = \nabla \times \frac{\partial V(r_{ij})}{\partial r_{ij}}[/tex]

In light of this equation however

[tex]\vec{F}_{ij} \cdot \hat{n} = \frac{\partial V(r_{ij})}{\partial r_{ij}}[/tex]

I could rewrite the Larmor energy as

[tex]\Delta H = \frac{2\mu}{\hbar Mc^2 e}(\vec{F}_{ij} \cdot \hat{n}) L \cdot S[/tex]

But I am sure many agree that is not very interesting.

(and this is really starting to make my brain cells burst), taking the curl of a force gives a F/length, however, the unit vector in this equation

[tex]\vec{F}_{ij} \cdot \hat{n} = \frac{\partial V(r_{ij})}{\partial r_{ij}}[/tex]

cancels these out and what I have again is the force again... yes? ... very circular... since this would be true, then we know what the force is anyway given earlier:

[tex]\vec{F}_{ij} = \frac{\partial V(r_{ij})}{\partial r_{ij}} \hat{n} [/tex]

Now, just a moment ago I found the Larmor energy written as

[tex]\Delta H = \frac{2\mu}{\hbar Mc^2 e} \frac{1}{r}\frac{\partial V(r_{ij})}{\partial r_{ij}} ( L \cdot S)[/tex]

Notice the 1/r term which is not in my original case. If that where true, and we plug in my force example again

[tex]\Delta H = \frac{2\mu}{\hbar Mc^2 e} \frac{1}{r}\frac{\partial V(r_{ij})}{\partial r_{ij}}\hat{n} ( L \cdot S)[/tex]

(If I am doing this right) the unit vector would cancel out with the radius term and what would be left with is

[tex]\Delta H = \frac{2\mu}{\hbar Mc^2 e} \frac{\partial V(r_{ij})}{\partial r_{ij}} ( L \cdot S)[/tex]

Now if that wasn't confusing I don't know what is.

I've either had it right from the beginning, or I have made a tiny mistake which is making a huge impact on my understanding of my own equation. Any help would be gladly appreciated.
 
Physics news on Phys.org
  • #2
What a mess my brain got into. Assuming this is correct mind you.

I found a solution to my problem (I think). The Larmor equation is

[tex]\Delta H = \frac{2\mu}{\hbar Mc^2 e} \frac{\partial V(r_{ij})}{\partial r_{ij}} ( L \cdot S)[/tex]

What I kept deriving was:

[tex]\vec{F}_{ij} \cdot \hat{n} = \frac{\partial V(r_{ij})}{\partial r_{ij}}[/tex]

What I really needed was the original derivation

[tex]\vec{F}_{ij} = \frac{\partial V(r_{ij})}{\partial r_{ij}} \hat{n}[/tex]

Then taking the curl of F gives

[tex]\nabla \times \vec{F}_{ij} = \frac{\partial V(r_{ij})}{\partial r_{ij}}[/tex]

Which removes the unit vector because nabla again is 1/length.

So what I think I have ended up with was the right derivation for the modified Larmor energy except for a dot product made on the unit vector after all, it would be

[tex]\Delta H = \frac{2\mu}{\hbar Mc^2 e} (\nabla \times \vec{F}_{ij}) L \cdot S[/tex]What a mess my brain got into. Assuming this is correct mind you.
 
  • #3
Now, does this seem right?
 

Related to Can you help me solve my mathematical problem?

1. How do I know if you can help me solve my mathematical problem?

As a scientist, I have a strong background in mathematics and problem-solving. I am confident in my ability to help you with your mathematical problem. However, I would need more information about the problem in order to determine if I can provide a solution.

2. What types of mathematical problems can you help me with?

As a scientist, I have experience with a wide range of mathematical problems, including algebra, calculus, geometry, statistics, and more. I am also familiar with various mathematical software and tools that can assist in solving problems.

3. Can you provide step-by-step instructions on how to solve my mathematical problem?

Yes, I can provide detailed instructions on how to solve your mathematical problem. However, keep in mind that every problem is unique and may require a different approach. I will do my best to explain the steps clearly and provide any necessary clarifications.

4. How long will it take for you to solve my mathematical problem?

The time it takes to solve a mathematical problem can vary depending on the complexity of the problem and the amount of information provided. I will work efficiently to solve the problem as quickly as possible, but it is difficult to give an exact time frame without more information.

5. Can you help me with my homework or exam questions?

I am happy to provide assistance and guidance with homework or exam questions related to mathematics. However, it is important to note that I will not provide direct answers or complete the work for you. I believe it is important for individuals to understand the process of problem-solving in order to truly learn and improve their skills in mathematics.

Similar threads

  • Quantum Physics
Replies
6
Views
1K
  • Quantum Physics
Replies
5
Views
626
Replies
3
Views
975
Replies
5
Views
465
  • Quantum Physics
Replies
29
Views
1K
  • Advanced Physics Homework Help
Replies
11
Views
1K
  • Quantum Physics
Replies
1
Views
628
Replies
17
Views
1K
  • Classical Physics
Replies
30
Views
2K
Back
Top